Reversing the Order of Double Integrals

  • Thread starter Thread starter hils0005
  • Start date Start date
  • Tags Tags
    Integration
Click For Summary
The discussion focuses on reversing the order of double integrals for a given domain defined by specific inequalities. Participants emphasize the importance of correctly identifying the limits of integration, noting that the outer integral must have constant limits rather than functions of y. A graphical representation is suggested to visualize the triangular area defined by the vertices (0, -1), (0, 1), and (1, 0). The need to separate the integral into two parts based on the triangular area is highlighted, with specific limits for each segment. The conversation concludes with a clarification of the intersection points and the correct formulation of the integrals.
hils0005
Messages
61
Reaction score
0
b]1. Homework Statement [/b]

Given Domain :\int\intf(x,y)dydx
0\leqx\leq1
x-1\leqy\leq2-2x
reiterate the integrals so the order is reversed

Homework Equations





The Attempt at a Solution


not really sure how to complete,
\int\intf(xy)dxdy
y+1\leqx\leq(2-y)/2
-1\leqy\leq2

Is this correct?
 
Physics news on Phys.org
hils0005 said:
b]1. Homework Statement [/b]

Given Domain :\int\intf(x,y)dydx
0\leqx\leq1
x-1\leqy\leq2-2x
reiterate the integrals so the order is reversed

Homework Equations





The Attempt at a Solution


not really sure how to complete,
\int\intf(xy)dxdy
y+1\leqx\leq(2-y)/2
-1\leqy\leq2
It should be obvious that it is not correct. The first integral is a number. This integral will, after integrating, be a function of y. In order to be a number, the limits of integration on the "outside" integeral, with respect to y, must be numbers, not functions of y.

Draw a picture! In the original "outside" integral, x ranges from 0 to -1. For each x, y ranges from x-1 up to 2- 2x. Those are straight lines and you should see that the area is a triangle with vertices (0, -1), (2, 0) and (0, 2).

Now look at it from the side. To cover that entire area, y needs to vary from -1 to 1: those will be the limits on the "outer", dy, integral. For each y, x varies form 0 up to x= a function of y, given by the line making the right boundary. It looks to me like you will need to separate that into two integrals: y form -1 to 0 and then from 0 to 1.
 
I have the verticies of the triangle at (2,0), (1,0), and (0,-1)?

wouldn't that mean dy would range from -1 to 2?

the lower part of the triangle dy from -1 to 0, x goes from 0 to y+1
ther upper part, dy ranges from 0 to 2, x goes from 0 to (y-2)/2

do you put an addition sign in between the the two integrals?
 
hils0005 said:
I have the verticies of the triangle at (2,0), (1,0), and (0,-1)?

wouldn't that mean dy would range from -1 to 2?

the lower part of the triangle dy from -1 to 0, x goes from 0 to y+1
ther upper part, dy ranges from 0 to 2, x goes from 0 to (y-2)/2

do you put an addition sign in between the the two integrals?
Then draw the graph more carefully. The two lines y= x- 1 and y= 2- 2x intersect at (1, 0). y= x-1 intersects x=0 at y= -1 and y= 2- 2x intersects x= 0 at y= 1. The vertices are, as I said, (0, -1), (0, 1), and (1, 0).
 
HallsofIvy said:
...and y= 2- 2x intersects x= 0 at y= 1...

Really? :wink:
 
Question: A clock's minute hand has length 4 and its hour hand has length 3. What is the distance between the tips at the moment when it is increasing most rapidly?(Putnam Exam Question) Answer: Making assumption that both the hands moves at constant angular velocities, the answer is ## \sqrt{7} .## But don't you think this assumption is somewhat doubtful and wrong?

Similar threads

  • · Replies 105 ·
4
Replies
105
Views
6K
  • · Replies 2 ·
Replies
2
Views
2K
Replies
4
Views
3K
Replies
2
Views
2K
  • · Replies 4 ·
Replies
4
Views
2K
  • · Replies 6 ·
Replies
6
Views
2K
  • · Replies 19 ·
Replies
19
Views
2K
  • · Replies 17 ·
Replies
17
Views
4K
  • · Replies 27 ·
Replies
27
Views
3K
  • · Replies 14 ·
Replies
14
Views
2K